the function g(x) = x2 is transformed to obtain function h: h(x) = g(x) + 1. Which statement describes how the graph of h is different from the graph of g? A. The graph of h is the graph of g horizontally shifted right 1 unit. B. The graph of h is the graph of g vertically shifted up 1 unit. C. The graph of h is the graph of g vertically shifted down 1 unit. D. The graph of h is the graph of g horizontally shifted left 1 unit.

Answers

Answer 1

The statement that  describes how the graph of h is different from the graph of g is: B. The graph of h is the graph of g vertically shifted up 1 unit.

Which statement describes how the graph of h is different from the graph of g?

The function h(x) = g(x) + 1 is obtained by adding a constant (1) to the output of the function g(x) = x^2. This means that the graph of h(x) will be the same as the graph of g(x), except that every point on the graph of h(x) will be shifted vertically upward by 1 unit.

Therefore, the correct statement is: B. The graph of h is the graph of g vertically shifted up 1 unit.

Learn more about function here:https://brainly.com/question/22340031

#SPJ1


Related Questions

The data set shown below represents the number of times some families went out for dinner the previous week. 4, 2, 2, 0, 1, 6, 3, 2, 5, 1, 2, 4, 0, 1 an unnumbered number line labeled numbers of dinners out. create a dot plot to represent the data. what can you conclude about the dot plot of the data set? check all that apply. the range of the number line should be 0 to 7 to represent the frequency. four families said they ate out twice the previous week. one family said they ate out 5 times the previous week. the data set is symmetrical. the median best represents the data set.

Answers

Answer: B, C, E

Step-by-step explanation: Other dude posted wrong answer.

HELPPP JUST 1 QUESTION!!! QUESTION IN PICTURE

Answers

Answer:

48.91

Step-by-step explanation:

r=cos^-1(.92)

r=23.07

cos(23.07)=45/y

y=45/cos(23.07)

48.91

Use the​ compound-interest formula to find the account balance​ A, where P is​ principal, r is interest​ rate, n is number of compounding periods per​ year, t is​ time, in​ years, and A is account balance. P r compounded t ​ $ ​% Daily

Answers

The account balance after 2 years is approximately $107.15.

What is the formula calculating  account balance A, given the principal P, interest rate r, number of compounding periods per year n, time t in years, and A is account balance when interest is compounded daily?

The compound interest formula is given by:

A = P * [tex](1 + r/n)^(^n^*^t^)[/tex]

Where:

P = Principal

r = Annual interest rate (as a decimal)

n = Number of times interest is compounded per year

t = Time in years

A = Final account balance

In this problem, we are given:

P = $100

r = 3.5% per year = 0.035 per year

n = 365 (since interest is compounded daily)

t = 2 years

Substituting these values in the formula, we get:

A = [tex]100 * (1 + 0.035/365)^(^3^6^5^*^2^)[/tex]

A ≈ $107.15

Learn more about compound

brainly.com/question/14117795

#SPJ11

Mark makes a pattern that starts with 5 and uses the rule "subtract 1, and then multiply by 3. " Which expression can be used to find the third number in Markâs pattern?

A. 5â1â3â1â3

B. 3(5â1)+3(5â1)

C. 3[3(5)â1]

D. 3[3(5â1)â1]


Choose one correct answer

Answers

The expression that can be used to find the third number in Mark's pattern is 3[3(5) - 1]. The correct option is C.

In Mark's pattern, the rule is to subtract 1 from the previous number and then multiply the result by 3.

Starting with 5 as the first number, we can apply this rule step by step to find the subsequent numbers.

First step: Subtract 1 from 5, giving us 4.

Second step: Multiply 4 by 3, which equals 12.

So, the second number in Mark's pattern is 12.

Now, to find the third number, we apply the same rule.

First step: Subtract 1 from 12, giving us 11.

Second step: Multiply 11 by 3, which equals 33.

Therefore, the third number in Mark's pattern is 33.

Option C, 3[3(5) - 1], correctly represents this calculation, where 5 is subtracted by 1, multiplied by 3, and then multiplied by 3 again.

To know more about Mark's pattern , refer here :

https://brainly.com/question/27421554#

#SPJ11

Use the method of Lagrange multipliers to find the points on the
curve x2 + y2 −6x + 7 = 0 that are closest to and furthest from the
point P = (0, 3).

Answers

Using the value of λ = (18 + √130)/18, we get: x = 3λ ≈ 4.895 y = 3λ - 3 ≈ 5.316 So the point on the curve that is furthest from P is approximately (4.895, 5.316).

To use the method of Lagrange multipliers, we first need to define our objective function and our constraint. Our objective function is the distance between the point P and a point on the curve, which can be expressed as:

f(x, y) = (x - 0)^2 + (y - 3)^2 = x^2 + (y - 3)^2

Our constraint is the equation of the curve:

g(x, y) = x^2 + y^2 - 6x + 7 = 0

To use the method of Lagrange multipliers, we need to introduce a new variable λ and solve the following system of equations:

∇f = λ∇g
g(x, y) = 0

where ∇f and ∇g are the gradients of f and g, respectively.

Taking the partial derivatives of f and g with respect to x and y, we have:

∂f/∂x = 2x
∂f/∂y = 2(y - 3)
∂g/∂x = 2x - 6
∂g/∂y = 2y

Setting ∇f equal to λ∇g, we have:

2x = λ(2x - 6)
2(y - 3) = λ(2y)

Simplifying these equations, we get:

x = 3λ
y = 3λ - 3

Substituting these expressions into the equation of the curve, we get:

(3λ)^2 + (3λ - 3)^2 - 6(3λ) + 7 = 0

Simplifying this equation, we get:

18λ^2 - 36λ + 13 = 0

Solving for λ, we get:

λ = (18 ± √130)/18

Substituting these values of λ into our expressions for x and y, we get the coordinates of the points on the curve that are closest to and furthest from the point P.

To find the point that is closest to P, we need to minimize the objective function f(x, y). Using the value of λ = (18 - √130)/18, we get:

x = 3λ ≈ 1.105
y = 3λ - 3 ≈ -0.316

So the point on the curve that is closest to P is approximately (1.105, -0.316).

To find the point that is furthest from P, we need to maximize the objective function f(x, y). Using the value of λ = (18 + √130)/18, we get:

x = 3λ ≈ 4.895
y = 3λ - 3 ≈ 5.316

So the point on the curve that is furthest from P is approximately (4.895, 5.316).

Learn more about point here:

brainly.com/question/30423121

#SPJ11

A project is budgeted for 1,200 hours and will last 6 weeks. a technician will be 25% billable to the project for the first three weeks and then 100% for the final three weeks. if a technician normally works 40 hours per week, how many total hours will the technician bill to the job?

Answers

The technician will bill a total of 150 hours to the job. To find the total hours, we need to determine how many hours the technician will work during the first three weeks and the last three weeks, and then add them together.


1. First three weeks:

The technician will be 25% billable during these weeks. They work 40 hours per week, so we need to calculate 25% of 40 hours for each week:


25% of 40 hours = 0.25 * 40 = 10 hours per week

Since there are three weeks, we'll multiply these hours by 3:
10 hours/week * 3 weeks = 30 hours


2. Last three weeks:

The technician will be 100% billable during these weeks. They work 40 hours per week, so they'll bill 40 hours for each of these weeks:
40 hours/week * 3 weeks = 120 hours


3. Finally,

we need to add the hours from the first and last three weeks together to find the total hours the technician will bill to the job:
30 hours (first three weeks) + 120 hours (last three weeks) = 150 hours


The technician will bill a total of 150 hours to the job.

To know more about  technician will bill refer here:

https://brainly.com/question/14290207#

#SPJ11

can someone pls help with this

Answers

A linear function would be the  best fit for the data.

A function that would be the best for this data is: D. y = -4/25(x) + 10

The amount of snow that would be on the ground when the temperature reaches 55° is 1.2 inches.

How to determine the line of best fit?

In this scenario, the temperature would be plotted on the x-axis (x-coordinate) of the scatter plot while the snow (inches) would be plotted on the y-axis (y-coordinate) of the scatter plot through the use of Microsoft Excel.

On the Microsoft Excel worksheet, you should right click on any data point on the scatter plot, select format trend line, and then tick the box to display an equation for the line of best fit (trend line) on the scatter plot.

From the scatter plot (see attachment) which models the relationship between the temperature and the snow (inches), an equation for the line of best fit is given by:

y = -0.16x + 10

y = -4/25(x) + 10

When x = 55, the amount of snow is given by;

y = -4/25(55) + 10

y = 1.2 inches.

Read more on scatter plot here: brainly.com/question/28605735

#SPJ1

Ray and Kelsey are working to graph a third-degree polynomial function that represents the first pattern in the coaster plan. Ray says the third-degree polynomial has four intercepts. Kelsey argues the function can have as many as three zeros only. Is there a way for the both of them to be correct? Explain your answer.


Kelsey has a list of possible functions. Pick one of the g(x) functions below and then describe to Kelsey the key features of g(x), including the end behavior, y-intercept, and zeros.
g(x) = (x + 2)(x − 1)(x − 2)
g(x) = (x + 3)(x + 2)(x − 3)
g(x) = (x + 2)(x − 2)(x − 3)
g(x) = (x + 5)(x + 2)(x − 5)
g(x) = (x + 7)(x + 1)(x − 1)

Answers

For the g(x) functions provided, here are their key features:

g(x) = (x + 2)(x − 1)(x − 2)

End behavior: As x approaches negative or positive infinity, g(x) approaches positive infinity.

Y-intercept: g(0) = -4

Zeros: x = -2, 1, 2

How to explain the function

Ray and Kelsey could both be accurate, all depending on the stated third-degree polynomial function.

It is conceivable for a third-degree polynomial to present up to three zeros, thus corroborating Kelsey's point that the function can have up to three intersection points with the x-axis maximum. Moreover, it can even occur that this function possesses a repeatable zero, causing a fourth interception.

Leans more about functions on

https://brainly.com/question/11624077

#SPJ1

Suppose M and C each represent the position number of a letter in the alphabet, but M represents the letters in the original message and C represents the letters in a secret code. The equation c=m+2 is used to encode a message.

Answers

The equation that can be used to decode the secret code is m = c - 2

How so you find the equation to decode the secret code?

For you to decode the secret message, you need to turn the the encoding process around. Find the inverse.

Since the encoding process uses the equation c = m + 2, to decode the message, all that need to be found is the value of m. This can be done by rearranging the encoding equation to solve for m

move 2 to c side. it becomes m = c-2

The above answer is in response to the full question below;

Suppose M and C each represent the position number of a letter in the alphabet, but M represents the letters in the original message and C represents the letters in a secret code. The equation c=m+2 is used to encode a message.

Write an equation that can be used to decode the secret code into the original message.

Find more exercises on decoding;

https://brainly.com/question/29976739

#SPJ1

2. an insurance salesman sells policies to 10 men, all of identical age and all of whom are in good health. according to his company's records, the probability that a man of this particular age will be alive in 20 years is 0.69. find the probability that in 20 years the number of the men that are still alive will be: a) exactly five b )more than 8 c)at least two ​

Answers

a) The probability that exactly five men will still be alive in 20 years is approximately 0.024.

b) The probability that more than eight men will still be alive in 20 years is approximately 0.057.

c) The probability that at least two men will still be alive in 20 years is approximately 0.999.

To calculate the probabilities, we can use the binomial distribution formula, where n is the number of trials, p is the probability of success, and x is the number of successes. Therefore,

a) P(X = 5) = (10 choose 5) * (0.69)⁵ * (0.31)⁵ ≈ 0.024

b) P(X > 8) = P(X = 9) + P(X = 10) = [(10 choose 9) * (0.69)⁹ * (0.31)¹] + [(10 choose 10) * (0.69)¹⁰ * (0.31)⁰] ≈ 0.057

c) P(X ≥ 2) = 1 - P(X = 0) - P(X = 1) = 1 - [(10 choose 0) * (0.69)⁰ * (0.31)¹⁰] - [(10 choose 1) * (0.69)¹ * (0.31)⁹] ≈ 0.999

In summary, we have used the binomial distribution formula to calculate the probability that exactly five men, more than eight men, and at least two men will still be alive in 20 years, given that the probability that a man of this particular age will be alive in 20 years is 0.69.

Learn more about probability

https://brainly.com/question/24756209

#SPJ4

Brooke and eileen are working on a math problem together and can't seem to agree on an answer. their teacher drew this number line on the board and asked them to think of a situation that could be represented by it.

brooke suggested the following situation:
christopher wants to buy a new bicycle and needs to earn more than $75 in order to have enough money.

eileen suggested the following situation:
paul is flying home from vacation and has less than 75 minutes left of the flight.

Answers

Both situations can be represented by the number line as they both involve values either greater than or less than 75.

The number line the teacher drew can represent both Brooke's and Eileen's situations.

In Brooke's situation, the number line can represent the amount of money Christopher needs to earn to buy a new bicycle. If he needs to earn more than $75, any point on the number line greater than 75 would represent the amount of money he has earned that is sufficient for purchasing the bicycle.

In Eileen's situation, the number line can represent the time left in Paul's flight. If Paul has less than 75 minutes left, any point on the number line less than 75 would represent the time remaining in his flight.

Both situations can be represented by the number line as they both involve values either greater than or less than 75.

Learn more about number line: https://brainly.com/question/13425491

#SPJ11

Solve the following quadratic function by utilizing the square root method. Y=xsquared minus nine

Answers

The solution of the quadratic equation is y = (x + 3)(x - 3).

What is the solution of the quadratic equation?

The solution of the quadratic equation is calculated by applying difference of two squares as shown below;

y = x² - 9

y = x² - 3²

the difference of two square of x² - 3² = (x + 3)(x - 3)

The solution of the quadratic equation is calculated as;

y = (x + 3)(x - 3)

Thus, solution of the quadratic equation has been determined using  square root method.

Learn more about difference of two squares here: https://brainly.com/question/3189867

#SPJ4

Chris bought 5 tacos and 2 burritos for $13. 25.


Brett bought 3 tacos and 2 burritos for $10. 75.


The price of one taco is $


The price of one burrito is $

Answers

If Chris bought 5 tacos and 2 burritos for $13. 25 and  Brett bought 3 tacos and 2 burritos for $10. 75, the price of one taco is $1.25, and the price of one burrito is $3.50.

Let the price of one taco be T and the price of one burrito be B. We have the following equations:

5T + 2B = $13.25
3T + 2B = $10.75

To find the prices of the taco and the burrito, we can use the system of equations. First, subtract the second equation from the first equation:

(5T + 2B) - (3T + 2B) = $13.25 - $10.75
2T = $2.50

Now, divide by 2 to find the price of one taco:

T = $1.25

Next, plug the value of T back into one of the equations (let's use the second equation):

3($1.25) + 2B = $10.75
$3.75 + 2B = $10.75

Now, subtract $3.75 from both sides:

2B = $7.00

Finally, divide by 2 to find the price of one burrito:

B = $3.50

So, the price of one taco is $1.25, and the price of one burrito is $3.50.

More on price: https://brainly.com/question/23747357

#SPJ11

Given the following exponential function, identify whether the change represents


growth or decay, and determine the percentage rate of increase or decrease.


y = 660(0. 902)

Answers

The function represents exponential decay with a percentage rate of decrease of 9.8%.

The given exponential function y = 660(0.902) represents decay because the base of the exponent is less than one.

This means that the output value of the function will decrease as the input value increases.

To determine the percentage rate of decrease, we need to find the value of the base of the exponent subtracted from one and then multiply it by 100.

The base of the exponent is 0.902, so we subtract it from one to get 0.098.

Multiplying by 100 gives us a percentage rate of decrease of 9.8%.

This means that for every unit increase in the input value, the output value of the function will decrease by approximately 9.8%.

For example, if the input value increases from 1 to 2, the output value will decrease by 9.8%, and if the input value increases from 2 to 3, the output value will again decrease by 9.8%.

Learn more about exponential decay here:

https://brainly.com/question/2193820

#SPJ4

Let w = 2xy + y2 - 4x2, += st, y=,= Compute Bu (1.-3) - 88 -(1, -3)

Answers

To compute Bu(1.-3) - 88 - (1, -3), we need to substitute the values of u and v into the expression for w.

First, we need to find the values of u and v. Since u = 1.-3 and v = (1, -3), we have:

u = 1.-3 = 1 - 0.3 = 0.7
v = (1, -3)

Next, we can substitute these values into the expression for w:

w = 2xy + y^2 - 4x^2
 = 2(1)(-3) + (-3)^2 - 4(1)^2  (substituting x = 1 and y = -3)
 = -6 + 9 - 4
 = -1

Finally, we can compute Bu(1.-3) - 88 - (1, -3) by multiplying the gradient of w by the vector (1, -3) and subtracting 88:

Bu(1.-3) - 88 - (1, -3) = (-8x + 2y, 2x + 2y)  (1, -3) - 88
                        = (-8(1) + 2(-3), 2(1) + 2(-3))  (1, -3) - 88
                        = (-14, -4)  (1, -3) - 88
                        = (-14)(1) + (-4)(-3) - 88
                        = -14 + 12 - 88
                        = -90

Therefore, Bu(1.-3) - 88 - (1, -3) = -90.

Since the question seems to have some typos or missing information, I'll assume you want to find the partial derivatives of w with respect to x and y, and evaluate them at the point (1, -3).

Given w = 2xy + y² - 4x², let's compute the partial derivatives:

∂w/∂x = 2y - 8x
∂w/∂y = 2x + 2y

Now, let's evaluate these partial derivatives at the point (1, -3):

∂w/∂x(1, -3) = 2(-3) - 8(1) = -6 - 8 = -14
∂w/∂y(1, -3) = 2(1) + 2(-3) = 2 - 6 = -4

Thus, the evaluated partial derivatives are ∂w/∂x(1, -3) = -14 and ∂w/∂y(1, -3) = -4.

Learn more about derivatives here: brainly.com/question/29096174

#SPJ11

What is 4x+2/39=5x-2/42?

Answers

We use the fundamental property of proportions where:

a/b = c/d if a d = b c

[tex] \space [/tex]

[tex] \bf \frac{4x + 2}{39} = \frac{5x - 2}{42} \\ \\ \bf 39 \cdot (5x - 2) = 42 \cdot (4x + 2) \\ \\ \bf 195x - 78 = 168x + 84 \\ \\ \bf 195x - 168x = 84 + 78 \\ \\ \bf 27x = 162 \\ \\ \bf x = \frac{162}{27} \implies \bf \red{ \boxed{ \bf x = 6} } [/tex]

The number is 6.

Hope that helps! Good luck! :)

The claim is that for 12 AM body​ temperatures, the mean is μ>98. 6°F. The sample size is n=8 and the test statistic is t= -2. 687



what is p value?

Answers

Value of p is approximately 0.987.

To find the p-value for the given claim that the mean body temperature at 12 AM is μ > 98.6°F with a sample size of n=8 and a test statistic of t=-2.687, follow these steps:

1. Identify the degrees of freedom: Since the sample size is n=8, the degrees of freedom (df) are calculated as n-1, which is 8-1=7.

2. Determine the tail of the test: The claim states that the mean body temperature is greater than 98.6°F (μ > 98.6), which indicates a right-tailed test.

3. Find the p-value using the t-distribution table or a calculator: With a test statistic of t=-2.687 and df=7, you can look up the corresponding p-value using a t-distribution table or an online calculator. Since it's a right-tailed test, the p-value will be the area to the right of the test statistic in the t-distribution.

After completing these steps, the p-value is found to be approximately 0.987.

Therefore, your answer is: The p-value for the claim that the mean body temperature at 12 AM is μ > 98.6°F, given a sample size of n=8 and a test statistic of t=-2.687, is approximately 0.987.

To know more about approximation refer here:

https://brainly.com/question/29267916?#

SPJ11

How can I get the answer for
A=
Vertex for y=

Answers

Answer:

1) a = 14

2) -4 (x - 2)² - 5

Step-by-step explanation:

To obtain a vertex, you take h and k in a equation.

So a(x-h)²+k = a(x-2)² -5

For the point (1, - 9),

a[(1)-2]² - 5 = - 9

a(1) = -9+5

a = -4

so the final equation is

-4(x-2)² - 5

I'm not 100% sure about this but I tried. Let me know if it makes sense

find the extremum of each function using the symmetry of its graph. Classify the etremum of the function as maximum or a minimum and state the of x at which it occurs k(x)(300+10x)(5-0.2x)

Answers

The extremum of the function is a minimum at x = -2.5

The given function is k(x)(300+10x)(5-0.2x).

To check for symmetry about the y-axis, we replace x with -x in the given function and simplify as follows:

k(-x)(300-10x)(5+0.2x)

To check for symmetry about the x-axis, we replace y with -y in the given function and simplify as follows:

k(x)(300+10x)(5-0.2x) = -k(x)(-300-10x)(5+0.2x)

To find these points, we set the function equal to zero and solve for x:

k(x)(300+10x)(5-0.2x) = 0

This equation has three solutions:

x = 0

x = -30

x = 25.

The midpoint of the line segment connecting these points is

(x1+x2) ÷ 2 = (-30+25) ÷ 2 = -2.5.

To determine the type of extremum at this point, we need to check the sign of the second derivative. The second derivative of the function is:

k(x)(-1200+x)(0.2x+15)

Since the function is symmetric about the x-axis, the second derivative will be negative at the extremum if it is maximum and positive if it is a minimum.

When x = -2.5, the second derivative is positive, which means that the function has a minimum at x = -2.5.

To learn more about extremum follow the link:

https://brainly.com/question/31343270

#SPJ1

The stem-and-leaf plot shows the number of push-ups done by each student in a Physical Education class. What is the mode of the number of push-ups?

Answers

The mode of the number of push-ups is 40.

What is the mode of the number of push-ups shown in the stem-and-leaf plot for a Physical Education class?

A stem-and-leaf plot is a way of organizing data where the stems (the tens digit) and leaves (the ones digit) are separated. Each row represents a stem and the leaves represent the values that belong to that stem.

Here's the stem-and-leaf plot for the number of push-ups:

3 | 5 6 8

4 | 0 0 1 2 2 3 5 6 8 9

5 | 0 1 3 4 5 5 7 8 9

6 | 0 1 2 2 3 4 5 7 8 9

7 | 0 2 5 8

8 | 1 2 4

9 | 0

To find the mode, we look for the value that appears most frequently. In this case, the number 40 appears three times, which is more than any other value. Therefore, the mode of the number of push-ups is 40.

Note that the stem-and-leaf plot makes it easy to see the distribution of the data. For example, we can see that there are a lot of values between 40 and 49, and relatively few values above 60.

We can also see that there are no values between 90 and 99.

Learn more about stem-and-leaf plot

brainly.com/question/12857419

#SPJ11

a) by using Venn-diagram. 75 students in a class like picnic or hiking or both. Out of them 10 like both the activities. The ratio of the number of students who like picnic to those who like hiking is 2 : 3. (i) Represent the above information in a Venn-diagram. (ii) Find the number of students who like picnic. (iii) Find the number of students who like hiking only. (iv) Find the percentage of students who like picnic only.​

Answers

(i) A Venn-diagram of this information is shown below.

(ii) The number of students who like picnic = 34

(iii) The number of students who like hiking only = 51

(iv) The percentage of students who like picnic only. = 45.33%

Let us assume that A represents the set of students who like picnic.

B represents the set of students who like the hiking.

The total number of students in a  class are: n(A U B) = 75

Out of 75 students, 10 like both the activities.

n(A ∩ B) = 10

The ratio of the number of students who like picnic to those who like hiking is 2 : 3

Let number of students like tea n(A) = 2x

and the number of students like coffee n(B) = 3x

n(A U B) = n(A) + n(B) - n(A ∩ B)

75 = 2x + 3x - 10

75 + 10 = 5x

85/5= x

x = 17

The number of students like picnic = 2x

                                                          = 2 × 17

                                                          = 34

The number of students like hiking = 3x

                                                           = 3 × 17

                                                           = 51

This informtaion in Venn diagram is shown below.

The percentage of students who like picnic only would be,

(34/75) × 100 = 45.33%

Learn more about the Venn diagram here:

https://brainly.com/question/30248978

#SPJ1

Create a story context for the following expressions ( 5 1/4 - 2 1/8) divided by 4 and 4 x ( 4. 8/0. 8)

Answers

To create a story context for the given expressions, which are (5 1/4 - 2 1/8) divided by 4 and 4 x (4.8/0.8).

Imagine there is a fruit store where you have to prepare fruit baskets for a local charity event. The first expression (5 1/4 - 2 1/8) divided by 4 can be a story about the number of apples to be distributed equally among four baskets.

You initially have 5 1/4 dozen apples, but you realize that 2 1/8 dozen of them are not suitable for the baskets.

To find out how many dozens of apples should be put into each basket, you need to subtract the unsuitable apples and divide the result by 4:

(5 1/4 - 2 1/8) / 4

Now, let's move on to the second expression, 4 x (4.8/0.8). This can be a story about the number of oranges you need to purchase for the fruit baskets. You already have 4.8 dozen oranges, but you need to add more to reach the desired ratio of oranges to apples.

Your friend suggests that for every 0.8 dozen oranges you currently have, you should add 4 more dozen oranges. To find out how many dozens of oranges you need to buy, you can use this formula:

4 x (4.8/0.8)

By creating these story contexts, you can use the given expressions to solve real-life problems, such as distributing fruits among charity baskets.

To know more about expressions refer here:

https://brainly.com/question/14083225

#SPJ11

The volume of this cube is 19,683 cubic yards. What is the value of s?​

Answers

The value of s is, 27 yards

:: Volume of cube with side s, is equal to s³

So, as the given volume is 19,683 cubic yards.

Therefore, it can related as,

s³ = 19,683 (yards)³

So,

s = ∛(19,683) yards

s = 27 yards

Read more about volume of cube at:

https://brainly.com/question/1972490

What are two algebraic expressions for the square root of x? (what are two ways of writing the square root of x?)​

Answers

The two algebraic expressions for the square root of x are x^(1/2) and √x.

An algebraic expression is an expression built up from constant algebraic numbers, variables, and the algebraic operations (addition, subtraction, multiplication, division and exponentiation by an exponent that is a rational number).

The square root function involves the square root symbol √ (which is read as "square root of"). The square root of a number 'x' is a number 'y' such that y2 = x. i.e., if y2 = x ⇒ y = √x. i.e., if 'x' is the square of 'y' then 'y' is the square root of 'x'.

There are two common ways to write the square root of x as an algebraic expression. The first way is to use fractional exponent notation, which is x^(1/2). The second way is to use radical notation, which is √x. Both of these expressions represent the square root of x in algebraic form.

Learn more about "square root": https://brainly.com/question/428672

#SPJ11

Will mark brainliest (to whoever explains this clearly)



Lizzie came up with a divisibility test for a certain number m that doesn't equal 1:



-Break a positive integer n into two-digit chunks, starting from the ones place. (For example, the number 354764 would break into the two-digit chunks 64, 47, 35. )



- Find the alternating sum of these two-digit numbers, by adding the first number, subtracting the second, adding the third, and so on. (In our example, this alternating sum would be 64-47+35=52. )



- Find m, and show that this is indeed a divisibility test for m (by showing that n is divisible by m if and only if the result of this process is divisible by m)

Answers

Lizzie's divisibility test states that a number n is divisible by a certain number m if and only if the alternating sum of its two-digit chunks is divisible by m.

How does Lizzie's divisibility test work?

Lizzie's divisibility test involves breaking a positive integer into two-digit chunks, finding the alternating sum of these chunks, and then determining if the result is divisible by a certain number m.

To apply the test:

Break the positive integer n into two-digit chunks from right to left.Calculate the alternating sum of these two-digit numbers, adding the first number, subtracting the second, adding the third, and so on.Find m, the divisor for which you want to test divisibility.If the result of the alternating sum is divisible by m, then n is also divisible by m.

To prove that this is a divisibility test for m, you need to show that n is divisible by m if and only if the result of the alternating sum is divisible by m.

Learn more about alternating

brainly.com/question/19718027

#SPJ11

Which list correctwhich list correctly identifies the steps to solving a word problem?ly identifies the steps to solving a word problem?

Answers

There is no one definitive list of steps to solving a word problem, as different types of problems may require different approaches.

However, a general set of steps that can be useful in solving many word problems is:

1. Read the problem carefully to understand what it is asking.

2. Identify the relevant information and the unknown quantity you need to find.

3. Translate the problem into an equation or set of equations that relate the given information to the unknown quantity.

4. Solve the equation(s) to find the value of the unknown quantity.

5. Check your answer to make sure it makes sense in the context of the problem.

Additional steps or variations on these steps may be necessary depending on the specific problem, but this general framework can be a useful starting point.

To know more about word problem refer here

https://brainly.com/question/29027588#

#SPJ11

Evaluate the integral dy (tan-'[y/8)) (64+y?) ( 1 + dy (tan-'(4/8)) (64+y?) =

Answers

Answer: ln|y/8| + C
Explanation:

First, we need to recognize that the derivative of arctan(x) is 1/(1+x^2). Therefore, the derivative of arctan(y/8) is 8/(64+y^2).

Now, using the substitution u = y/8, we can rewrite the integral as:

∫(1/u)(64+64u^2)(8/(64+64u^2))du

Simplifying, we get:

∫(1/u)du = ln|u| = ln|y/8|

Therefore, the final answer is:

ln|y/8| + C

where C is the constant of integration.

A movie theater has a seating capacity of 349. The theater charges $5. 00 for children, $7. 00 for students, and $12. 00 for adults. There are half as many adults as there are children. If the total ticket sales was $ 2540, How many children, students, and adults attended?

Answers

194 children, 58 students, and 97 adults attended the movie.

Let's use algebra to solve this problem.

Let's assume the number of children who attended the movie is C, the number of students is S, and the number of adults is A.

From the problem, we know that:

The seating capacity of the theater is 349:

C + S + A = 349

The theater charges $5 for children, $7 for students, and $12 for adults:

5C + 7S + 12A = $2540

There are half as many adults as there are children:

A = 1/2C

Now we can substitute A = 1/2C from the third equation into the first and second equations:

C + S + 1/2C = 349

3/2C + S = 349

5C + 7S + 12(1/2C) = $2540

5C + 7S + 6C = $2540

11C + 7S = $2540

Now we have two equations with two variables, C and S.

We can solve for S in the first equation:

3/2C + S = 349

S = 349 - 3/2C

Now we can substitute S = 349 - 3/2C into the second equation:

11C + 7S = $2540

11C + 7(349 - 3/2C) = $2540

11C + 2443 - 10.5C = $2540

0.5C = 97

C = 194

Therefore, 194 children attended the movie of total sales.

We can use A = 1/2C from the third equation to find the number of adults:

A = 1/2C

A = 1/2(194)

A = 97

Therefore, 97 adults attended the movie.

We can use C + S + A = 349 to find the number of students:

C + S + A = 349

194 + S + 97 = 349

S = 58

Therefore, 58 students attended the movie.

In summary, 194 children, 58 students, and 97 adults attended the movie.

To learn more about sales, click here:

https://brainly.com/question/29857652

#SPJ11

Catering company provides packages for weddings and for showers. The cost per person for small groups is


pproximately Normally distributed for both weddings and showers. The mean cost for weddings is $82. 30 with a


andard deviation of $18. 20, while the mean cost for showers is $65 with a standard deviation of $17. 73. If 9


eddings and 6 showers are randomly selected, what is the probability the mean cost of the weddings is more than


e mean cost of the showers?

Answers

The probability that the mean cost of the 9 weddings is more than the mean cost of the 6 showers is approximately 0.0207 or 2.07%.

The probability that the mean cost of the 9 weddings is more than the mean cost of the 6 showers can be found using the Z-score and the difference between the means of two normally distributed variables.

1: Calculate the difference in means and standard deviations.
Δμ = μ_weddings - μ_showers = $82.30 - $65 = $17.30
Δσ = sqrt((σ_weddings²/n_weddings) + (σ_showers²/n_showers)) = sqrt((18.20²/9) + (17.73²/6)) = $8.47

2: Calculate the Z-score.
Z = (Δμ - 0) / Δσ = (17.30 - 0) / 8.47 ≈ 2.04

3: Determine the probability using a Z-table.
P(Z > 2.04) ≈ 0.0207

To know more about  Z-score click on below link:

https://brainly.com/question/15016913#

#SPJ11

AREA OF TRAPEZOID PLS ANSWER ASAP

Answers

So first U solve the triangle then once you get that you multiply it by the square
Other Questions
Complete the word equation for making a salt. Metal oxide + salt + water We will use boosting to predict Salary in the Hitters data set. A) Remove the observations for whom the salary information is unknown, and then log-transform the salaries. B) Create a training set consisting of the first 200 observations, and a test set consisting of the remaining observations. C) Perform boosting on the training set with 1,000 trees. What is the the test set MSE Write an eassy on the causes and effects of bunking classes in approximately in 190 words which of the following strategies most likely could improve the response rate of the confirmation of accounts receivable? including a list of items or invoices that constitute the account balance requesting customers to respond to the confirmation requests directly to the auditor by fax or e-mail restricting the selection of accounts to be confirmed to those customers with relatively large balances notifying the recipients that second requests will be mailed if they fail to respond in a timely manner question Which of the following shows an example of the identity property of 0? + (-2) = 0O 0+ (-10%) = - 10/0 + 1 = 1/20-3 1/2+7= 3/1/ 3. Through this coup, code-named Operation TP-Ajax, Mossadegh was deposed and ___________________________________________________. The new leader was a member of Irans royal family named ______________________________. The Shahs government was ________________________________________. In exchange for tens of millions of dollars in foreign aid, he returned 80 percent of Irans oil reserves to the ______________________________________________________ What is the money multiplier when the reserve requirement is:(Instructions: Enter your responses rounded to three decimal places.)(a) 0.040?(b) 0.125?(c) 0.400?(d) 0.200? Kelly has a magic pot which can slowly cook one ingredient into others. She already mastered the cooking techniques and concludes that the process can be modeled using markov chain represented by a transition matrix A. You are given the ingredient list material_list = ["snake oil", "silver", "caviar", "bones", "uranium", "plutonium"] which corresponds to the rows and columns order of A. Each entry Aij of A represents the fraction of the jt ingredient of material_list that will turn into the the ingredient per minute without losing any mass. One day Kelly started making some of her magical soup and after a few minutes she realized she forgot to start the timer. She could not remember the time she started cooking, or even the ingredient composition of her initial soup. She can only recall that at the beginning of the cooking, (35+ 0. 1)% of the mass of all materials is snake oil. In fear of ruining her precious ingredients, she immediately casts a spell to determine the current composition of the soup and stores it in the vector comp, where each entry is the fraction in terms of mass of each ingredient, following the order in material_list. Write a code snippet to help Kelly with the following: 1. Use the current ingredient composition given in comp to determine the initial ingredient composition. Store in the variable initial_frac the initial fraction of the ingredient bones. 2. Suppose Kelly decides to leave the soup cooking for a very long time, until the ingredient composition in the pot is no longer changing. Help her determine this final ingredient composition and store it in the vector final_comp, where each entry is the fraction in terms of mass of each ingredient, following the order in material_list. The setup code will provide the following variables: The setup code will provide the following variables: A Name Type Description 2d numpy array transition matrix material_list Python list names of the ingredients comp 1d numpy array composition of the ingredients in the pot Your code snippet should define the following variables: Name initial_frac final_comp Type float 1d numpy array Description initial fraction of bones in terms of mass final ingredient composition hich firm is experiencing diseconomies of scale? a. firm b only b. firm c only c. firm a only d. firm a and firm b only You have decided to purchase a car for $25,625. The credit union requires a 10% down payment and will finance the balance with a 9% annual interest loan for 36 months. The sales tax in your city is 7. 5%, and the license and title charges are $175. 13. What is the total purchase price of the car including tax, license, and title? Round your answer to the nearest cent. A. $24,949. 80 c. $27,722. 01 b. $24,967. 32 d. $27,735. 14. What is the volume of a container of 4 moles of gas at 200k with a pressure of 3 atm Management at Reilly Company is considering the purchase of a machine that would cost $240,000 and would last for 10 years. At the end of 10 years, the machine would have a salvage value of $21,500. By reducing labor and other operating costs, the machine would provide annual cost savings of $37,000. The company requires a minimum pretax return of 10% on all investment projects. (Ignore income taxes. ) Click here to view Exhibit 14B-1 and Exhibit 14B-2, to determine the appropriate discount factor(s) using the tables provided. The net present value of the proposed project is closest to: 4 Gabriela is building a wooden box with a rectangular base that is 18 in. By 15 in. and is 15 in. Tall. Part AIf she wants an open box without a top, how much wood will Gabriela use?Strow your work brandy has a rectangular wooden deck that measures 7 feet by 12 feet she builds an addition to the deck that is 4 feet longer. what is the perimeter of the deck now Across: 6. It is a type of position in those poles or garters are separated outside the tappers' knees. Down:4. Movement of the feet describe as a spring on one foot and landing on the same foot. 9. The step pattern of this movement is the step, close, step[tex] \\ \\ [/tex](this crossword puzzle is abt tinikling dance) Ellen mixed 1over 4 kg of flour with 2 over 9 kg of sugar. Determine a reasonable estimate for the amount of flour and sugar combined The table gives the average per capita income, d, in a region of the country as a function of the percent unemployed, u.Which equation represents this data algebraically? a farmer made a loss of 28% by selling a gold for1440shillings what percentage profit would have made if he had sold the goat.for.sh 2100 Write a letter to your friend advising him/her not to be part of any conflicts and tell him/her about its consequences to. Explain how your model is different from the model in the picture.